LSAT and Law School Admissions Forum

Get expert LSAT preparation and law school admissions advice from PowerScore Test Preparation.

 Administrator
PowerScore Staff
  • PowerScore Staff
  • Posts: 8917
  • Joined: Feb 02, 2011
|
#37405
Complete Question Explanation

Flaw in the Reasoning. The correct answer choice is (C)

The stimulus contains a predictable argument/counterargument structure, introducing the anticonclusion
by describing what “some critics claim.” Regardless of what the critics might say, the
concert promoter is convinced that their concert series have popular appeal. Why? Because they
made just as much money from the sales of memorabilia at these concerts as they did from similar
sales at comparable series. She is assuming, of course, that the “comparable concerts” were popular
enough to compare their income to the income from the concerts in question. If that were not the
case, then the “comparable concerts” would be an exceedingly poor benchmark against which to
measure the success of any concert series.

You should immediately recognize this as a Relativity Flaw. Test makers often play on the distinction
between relative states and absolute states within arguments. This distinction is often subtle and can
be difficult to spot. The premises here only establish a comparison between the incomes from similar
sales at comparable concerts; meanwhile, the conclusion is an absolute statement, arguing that a
particular concert series has popular appeal.

Answer choice (A): The author does not attack the critics on the basis of emotional considerations.
There is no Appeal to Emotion flaw in this argument.

Answer choice (B): This is an incredibly attractive, yet incorrect, answer choice. Since the phrase
“takes for granted” is used to describe an unwarranted assumption in Flaw questions, we can apply
the Assumption Negation Technique to see if the logical opposite of the alleged assumption weakens
the argument. So, what if the income from sales of memorabilia were not the only indicator of
popular appeal? Even if that were the case, it is unclear whether the other indicators in question
would produce different results from the ones described in the argument. They might not. If they
were to show that the concert series are popular, this would strengthen—not weaken—the author’s
argument. Furthermore, it is also possible that the other indicators of popular appeal are not as
reliable or easily measurable as the sales of memorabilia.

Think of it in this way: even if the LSAT were not the only way to measure one’s potential to excel
in law school (it isn’t), it may still be a reliable indicator of potential for success. Since the logical
opposite of the assumption described in answer choice (B) does not weaken the concert promoter’s
conclusion, this answer choice does not describe an assumption upon which the argument is based.

Answer choice (C): This is the correct answer choice. If the comparable series did not possess
popular appeal, then they would provide a poor benchmark against which to measure the popular
appeal of the concerts in question. Since the logical opposite of the statement described in answer
choice (C) weakens the argument, that statement is an assumption upon which the argument
depends.

Of course, the best way to approach this question would be to prephrase the Relativity Flaw and
look for an answer properly describing it. Failing that, you can logically negate any answer choice
that begins with the phrase “takes for granted,” or “presumes without justification.” If the answer
choice indeed contains a statement that the author took for granted (i.e. assumed), its logical opposite
should weaken the conclusion of the argument.

Answer choice (D): Given that the benchmark concert series are described as “comparable” and their
sales—“similar,” there is no reason to suspect that the author draws a conclusion about the popularity
of a series based on a comparison with other, dissimilar events.

Answer choice (E): There is no evidence of an Error of Division or Composition in this argument.
 agroves
  • Posts: 32
  • Joined: Aug 03, 2013
|
#10906
Hello again,

Can you please explain why answer choice C is a better answer than B? I would also appreciate if you could walk me through your though process of arriving at C.

Thank you!

Angela
 Steve Stein
PowerScore Staff
  • PowerScore Staff
  • Posts: 1153
  • Joined: Apr 11, 2011
|
#10921
Hi,

In many cases, one word can make the difference; in this example, take another look at answer choice B: does the concert promoter take for granted that memorabilia-based income is the only indicator? No. But the promoter does try to prove the series' popularity by comparing it to others. Without knowing anything about the others, though, this really isn't reliable proof (...maybe none of the comparable series are popular either, in which case outpacing their sales would not be such an impressive feat).

I hope that's helpful! Please let me know whether this is clear--thanks!

~Steve
 agroves
  • Posts: 32
  • Joined: Aug 03, 2013
|
#10929
Thanks Steve! That was my hunch when I looked at it a second time.

Appreciate the help!
 Basia W
  • Posts: 108
  • Joined: Jun 19, 2014
|
#15719
Good evening,

I chose b) in relation to this question- would this answer be incorrect because it says "sole indicator"? I guess I was lulled by the stimulus because it said: "sales at comparable series" so maybe this was the assumption that was more needed...

thank you,

best,

Basia
 Lucas Moreau
PowerScore Staff
  • PowerScore Staff
  • Posts: 216
  • Joined: Dec 13, 2012
|
#15746
Hello, Basia,

The meat of the promoter's argument is "Our income from memorabilia sales, etc, is at least as good as that for similar series. Therefore, we do not lack popular appeal."

This only follows if matching or exceeding the income from memorabilia sales of those similar ones means that it must be true that a concert series has popular appeal. But this is never proven!

It wouldn't prove anything if all the similar series themselves lacked popular appeal...you can't prove that something has popular appeal by showing that it's the same as something that doesn't have popular appeal.

B is wrong because it doesn't quite attack the unproven assumption that is the main flaw of the argument.

Hope that helps,
Lucas Moreau
 LsatSoon2BMaster
  • Posts: 7
  • Joined: Oct 27, 2015
|
#20583
I have a big problem with this question. I chose D. I watched a video explanation for the question but his reasoning infers that it is known sales are correlated with popular appeal. To me this is not always the case so I thought it would be unreasonable to assume that. For example my schools parking department may generate a lot of sales of parking tickets but to most they still lack popular appeal. I know that is a poor example but still I do not understand how you can infer that concert t-shirt sales are correlated with popular appeal.

Now, I chose D.

I think he draws a conclusion on popular appeal by comparison with other dissimilar events (t-shirt sales).

The video says he is comparing his concert to similar events which is why this is wrong. While he is comparing the concert to other similar concerts, he is also comparing popular appeal with t-shirt sales which I found to be dissimilar.


Sorry that was a mouthful but here are my questions:

1. Why is it reasonable to infer t-shirt sales correlate with popularity?
2. Why is what I say about D is wrong? It seems there are two comparisons making D correct or wrong depending on how you look at it. (I didn't even think about the second way until I watched the video)

Thanks in advanced for any help!
 Anthony Esposito
PowerScore Staff
  • PowerScore Staff
  • Posts: 14
  • Joined: Sep 16, 2015
|
#20602
Hi LSAT Master,

Love the handle! That shows the positive thinking necessary to do your best on test day.

Let's take a look at why Answer Choice (C) is correct.

The concert promoter is trying to argue what? Well, that the critics are mistaken. What are they mistaken about? Well, that his/her concert series lacks popular appeal. So, the concert promoter is arguing that the concert series has popular appeal.

Ok, then, what's his or her evidence that the concert series has popular appeal? Well, the income from his/her concert series beat out some other comparable concert series. The problem with the concert promoter's logic is we don't know anything about the comparable concert series' popular appeal.

If the comparable concert series that was beaten out in sales is just awful and nobody likes it, then we cannot say for certain that the first concert promoter's concert series has popular appeal. We just know it gets better income than an awful concert series that definitely does not have popular appeal.

Regarding why Answer Choice (D) is incorrect, the other concert series isn't described as dissimilar in the stimulus. In fact, it is called a "comparable series." And, remember, the first concert promoter's concert series could be more popular than the other comparable concert series, but that doesn't necessarily mean it has "popular appeal," just that it's more popular than than the comparable concert series.

Keep it up and you'll be on your way to LSAT Master status soon enough!

Hope this helps,
Anthony
 TOgren2424
  • Posts: 17
  • Joined: May 21, 2017
|
#35328
Would B be correct if it stated "takes for granted that income from sales of memorabilia is an indicator of popular appeal."

In essence the only thing that makes it wrong is the "only" in the answer choice?

Don't know how i missed the "only." I'll write it off to being tired.
 Emily Haney-Caron
PowerScore Staff
  • PowerScore Staff
  • Posts: 577
  • Joined: Jan 12, 2012
|
#35386
Hi TOgren2424,

Yep, you're exactly correct! The two things that cause little errors like that are being in too much of a hurry (particularly if you are doing the question or section timed) or, as you pointed out, being tired. Your reasoning is spot-on, though, so you're on the right track!

Get the most out of your LSAT Prep Plus subscription.

Analyze and track your performance with our Testing and Analytics Package.